Difference between revisions of "2018 USAMO Problems/Problem 3"

(Created page with "==Problem 3== For a given integer <math>n\ge 2,</math> let <math>\{a_1,a_2,…,a_m\}</math> be the set of positive integers less than <math>n</math> that are relatively prime...")
 
(Solution)
 
(4 intermediate revisions by 2 users not shown)
Line 3: Line 3:
  
  
==Solution==
+
==Solution (NO FORMATTING)==
 +
https://maa.org/sites/default/files/pdf/AMC/usamo/2018/2018USAMO.pdf
 +
The integer m in the statement of the problem is ϕ(n), where ϕ is the Euler totient function.
 +
Throughout our proof we write p
 +
s
 +
|| m, if s is the greatest power of p that divides m.
 +
We begin with the following lemma:
 +
Lemma 1. If p is a prime and p
 +
s divides n for some positive integer s, then 1k + 2k + · · · + n
 +
k
 +
is
 +
divisible by p
 +
s−1
 +
for any integer k ≥ 1.
 +
2018 USAMO – Solutions 4
 +
Proof. Let {a1, a2, . . . , am} be a complete reduced residue set modulo p
 +
s and m = p
 +
s−1
 +
(p − 1).
 +
First we prove by induction on s that for any positive integer k, a
 +
k
 +
1 + a
 +
k
 +
2 + · · · + a
 +
k
 +
m is divisible by
 +
p
 +
s−1
 +
. The base case s = 1 is true. Suppose the statement holds for some value of s. Consider the
 +
statement for s + 1. Note that
 +
{a1, . . . , am, ps + a1, . . . , ps + am, . . . , ps
 +
(p − 1) + a1, . . . , ps
 +
(p − 1) + am}
 +
is a complete reduced residue set modulo p
 +
s+1. Therefore, the desired sum of k-th powers is equal
 +
to
 +
a
 +
k
 +
1 + · · · + a
 +
k
 +
m + · · · + (p
 +
s
 +
(p − 1) + a1)
 +
k + · · · + (p
 +
s
 +
(p − 1) + am)
 +
k ≡ p(a
 +
k
 +
1 + · · · + a
 +
k
 +
m) ≡ 0 (mod p
 +
s
 +
),
 +
where we have used the induction hypothesis for the second congruence. This gives the induction
 +
step.
 +
Now we are ready to prove the lemma. Because numbers from 1 to n can be split into blocks of
 +
consecutive numbers of length p
 +
s
 +
, it is enough to show that 1k + 2k +· · · + (p
 +
s
 +
)
 +
k
 +
is divisible by p
 +
s−1
 +
for any positive integer k. We use induction on s. The statement is true for s = 1. Assume the
 +
statement is true for s − 1. The sum
 +
1
 +
k + 2k + · · · + (p
 +
s
 +
)
 +
k = a
 +
k
 +
1 + a
 +
k
 +
2 + · · · + a
 +
k
 +
m + p
 +
k
 +
 +
1
 +
k + 2k + · · · + (p
 +
s−1
 +
)
 +
k
 +
 +
is divisible by p
 +
s−1
 +
, because p
 +
s−1
 +
| a
 +
k
 +
1 + · · · + a
 +
k
 +
m and by the induction hypothesis p
 +
s−2
 +
| 1
 +
k + · · · +
 +
(p
 +
s−1
 +
)
 +
k
 +
.
 +
Now we proceed to prove a second lemma, from which the statement of the problem will immediately
 +
follow:
 +
Lemma 2. Suppose p is a prime dividing n. Let {a1, . . . , am} be a complete reduced residue set
 +
mod n, and define s by p
 +
s
 +
|| m. Then p
 +
s divides a
 +
k
 +
1 + · · · + a
 +
k
 +
m for any integer k ≥ 1.
 +
Proof. We fix p, and use induction on the number of prime factors of n (counted by multiplicity)
 +
that are different from p. If there are no prime factors other than p, then n = p
 +
s+1
 +
, m = p
 +
s
 +
(p − 1),
 +
and we proved in Lemma 1 that a
 +
k
 +
1 +· · ·+a
 +
k
 +
m is divisible by p
 +
s
 +
. Now suppose the statement is true
 +
for n. We show that it is true for nq, where q is a prime not equal to p.
 +
Case 1. q divides n. We have p
 +
s
 +
|| ϕ(n) and p
 +
s
 +
|| ϕ(nq), because ϕ(nq) = qϕ(n). If {a1, a2, . . . , am}
 +
is a complete reduced residue set modulo n, then
 +
{a1, . . . , am, n + a1, . . . , n + am, . . . , n(q − 1) + a1, . . . , n(q − 1) + am}
 +
is a complete reduced residue set modulo nq. The new sum of k-th powers is equal to
 +
a
 +
k
 +
1 + · · · + a
 +
k
 +
m + · · · + (n(q − 1) + a1)
 +
k + · · · + (n(q − 1) + am)
 +
k = mnk
 +
 +
1
 +
k + · · · + (q − 1)k
 +
 +
+
 +
2018 USAMO – Solutions 5
 +
 +
k
 +
1
 +
 +
n
 +
k−1
 +
 +
1
 +
k−1 + · · · + (q − 1)k−1
 +
 +
(a1 + · · · + am) + · · · + q(a
 +
k
 +
1 + · · · + a
 +
k
 +
m).
 +
This sum is divisible by p
 +
s because p
 +
s
 +
|| m and p
 +
s
 +
| a
 +
j
 +
1 + a
 +
j
 +
2 + · · · + a
 +
j
 +
m for any positive integer j.
 +
Case 2. q doesn’t divide n. Suppose p
 +
b
 +
|| q − 1, where b ≥ 0. Note that ϕ(nq) = ϕ(n)(q − 1), so
 +
p
 +
s
 +
|| ϕ(n) and p
 +
s+b
 +
|| ϕ(nq). Let {a1, . . . , am} be a complete reduced residue set modulo n. The
 +
complete reduced residue set modulo nq consists of the mq numbers
 +
{a1, . . . , am, n + a1, . . . , n + am, . . . , n(q − 1) + a1, . . . , n(q − 1) + am}
 +
with the m elements {qa1, qa2, . . . , qam} removed.
 +
The new sum of k-th powers is equal to
 +
a
 +
k
 +
1 + · · · + a
 +
k
 +
m + · · · + (n(q − 1) + a1)
 +
k + · · · + (n(q − 1) + am)
 +
k − q
 +
k
 +
(a
 +
k
 +
1 + · · · + a
 +
k
 +
m) =
 +
mnk
 +
 +
1
 +
k + · · · + (q − 1)k
 +
 +
+
 +
 +
k
 +
1
 +
 +
n
 +
k−1
 +
 +
1
 +
k−1 + · · · + (q − 1)k−1
 +
 +
(a1 + · · · + am) + · · ·
 +
· · · +
 +
 +
k
 +
k − 1
 +
 +
n (1 + · · · + (q − 1)) (a
 +
k−1
 +
1 + · · · + a
 +
k−1
 +
m ) + q(a
 +
k
 +
1 + · · · + a
 +
k
 +
m) − q
 +
k
 +
(a
 +
k
 +
1 + · · · + a
 +
k
 +
m).
 +
Each term
 +
 +
k
 +
j
 +
 +
n
 +
k−j
 +
 +
1
 +
k−j + · · · + (q − 1)k−j
 +
 +
(a
 +
j
 +
1 + · · · + a
 +
j
 +
m),
 +
for 0 ≤ j ≤ k − 1, is divisible by p
 +
s+b because p | n
 +
k−j
 +
, p
 +
s
 +
| a
 +
j
 +
1 + · · · + a
 +
j
 +
m, and p
 +
b−1
 +
| 1
 +
k−j + · · · +
 +
(q − 1)k−j by Lemma 1.
 +
Also (q
 +
k − q)(a
 +
k
 +
1 + · · · + a
 +
k
 +
m) is divisible by p
 +
s+b because p
 +
b
 +
| q − 1 | q
 +
k − q and p
 +
s
 +
| a
 +
k
 +
1 + · · · + a
 +
k
 +
m.
 +
Thus p
 +
s+b divides our sum and our proof is complete.
 +
Remark. In fact, one can also show the converse statement: if {a1, a2, . . . , am} is as defined in
 +
the problem and a
 +
k
 +
1 + a
 +
k
 +
2 + · · · + a
 +
k
 +
m is divisible by m for every positive integer k, then every prime
 +
that divides m also divides n.
 +
{{MAA Notice}}
 +
 
 +
{{USAMO newbox|year=2018|num-b=2|num-a=4}}

Latest revision as of 22:00, 11 February 2024

Problem 3

For a given integer $n\ge 2,$ let $\{a_1,a_2,…,a_m\}$ be the set of positive integers less than $n$ that are relatively prime to $n.$ Prove that if every prime that divides $m$ also divides $n,$ then $a_1^k+a_2^k + \dots + a_m^k$ is divisible by $m$ for every positive integer $k.$


Solution (NO FORMATTING)

https://maa.org/sites/default/files/pdf/AMC/usamo/2018/2018USAMO.pdf The integer m in the statement of the problem is ϕ(n), where ϕ is the Euler totient function. Throughout our proof we write p s || m, if s is the greatest power of p that divides m. We begin with the following lemma: Lemma 1. If p is a prime and p s divides n for some positive integer s, then 1k + 2k + · · · + n k is divisible by p s−1 for any integer k ≥ 1. 2018 USAMO – Solutions 4 Proof. Let {a1, a2, . . . , am} be a complete reduced residue set modulo p s and m = p s−1 (p − 1). First we prove by induction on s that for any positive integer k, a k 1 + a k 2 + · · · + a k m is divisible by p s−1 . The base case s = 1 is true. Suppose the statement holds for some value of s. Consider the statement for s + 1. Note that {a1, . . . , am, ps + a1, . . . , ps + am, . . . , ps (p − 1) + a1, . . . , ps (p − 1) + am} is a complete reduced residue set modulo p s+1. Therefore, the desired sum of k-th powers is equal to a k 1 + · · · + a k m + · · · + (p s (p − 1) + a1) k + · · · + (p s (p − 1) + am) k ≡ p(a k 1 + · · · + a k m) ≡ 0 (mod p s ), where we have used the induction hypothesis for the second congruence. This gives the induction step. Now we are ready to prove the lemma. Because numbers from 1 to n can be split into blocks of consecutive numbers of length p s , it is enough to show that 1k + 2k +· · · + (p s ) k is divisible by p s−1 for any positive integer k. We use induction on s. The statement is true for s = 1. Assume the statement is true for s − 1. The sum 1 k + 2k + · · · + (p s ) k = a k 1 + a k 2 + · · · + a k m + p k � 1 k + 2k + · · · + (p s−1 ) k � is divisible by p s−1 , because p s−1 | a k 1 + · · · + a k m and by the induction hypothesis p s−2 | 1 k + · · · + (p s−1 ) k . Now we proceed to prove a second lemma, from which the statement of the problem will immediately follow: Lemma 2. Suppose p is a prime dividing n. Let {a1, . . . , am} be a complete reduced residue set mod n, and define s by p s || m. Then p s divides a k 1 + · · · + a k m for any integer k ≥ 1. Proof. We fix p, and use induction on the number of prime factors of n (counted by multiplicity) that are different from p. If there are no prime factors other than p, then n = p s+1 , m = p s (p − 1), and we proved in Lemma 1 that a k 1 +· · ·+a k m is divisible by p s . Now suppose the statement is true for n. We show that it is true for nq, where q is a prime not equal to p. Case 1. q divides n. We have p s || ϕ(n) and p s || ϕ(nq), because ϕ(nq) = qϕ(n). If {a1, a2, . . . , am} is a complete reduced residue set modulo n, then {a1, . . . , am, n + a1, . . . , n + am, . . . , n(q − 1) + a1, . . . , n(q − 1) + am} is a complete reduced residue set modulo nq. The new sum of k-th powers is equal to a k 1 + · · · + a k m + · · · + (n(q − 1) + a1) k + · · · + (n(q − 1) + am) k = mnk � 1 k + · · · + (q − 1)k � + 2018 USAMO – Solutions 5 � k 1 � n k−1 � 1 k−1 + · · · + (q − 1)k−1 � (a1 + · · · + am) + · · · + q(a k 1 + · · · + a k m). This sum is divisible by p s because p s || m and p s | a j 1 + a j 2 + · · · + a j m for any positive integer j. Case 2. q doesn’t divide n. Suppose p b || q − 1, where b ≥ 0. Note that ϕ(nq) = ϕ(n)(q − 1), so p s || ϕ(n) and p s+b || ϕ(nq). Let {a1, . . . , am} be a complete reduced residue set modulo n. The complete reduced residue set modulo nq consists of the mq numbers {a1, . . . , am, n + a1, . . . , n + am, . . . , n(q − 1) + a1, . . . , n(q − 1) + am} with the m elements {qa1, qa2, . . . , qam} removed. The new sum of k-th powers is equal to a k 1 + · · · + a k m + · · · + (n(q − 1) + a1) k + · · · + (n(q − 1) + am) k − q k (a k 1 + · · · + a k m) = mnk � 1 k + · · · + (q − 1)k � + � k 1 � n k−1 � 1 k−1 + · · · + (q − 1)k−1 � (a1 + · · · + am) + · · · · · · + � k k − 1 � n (1 + · · · + (q − 1)) (a k−1 1 + · · · + a k−1 m ) + q(a k 1 + · · · + a k m) − q k (a k 1 + · · · + a k m). Each term � k j � n k−j � 1 k−j + · · · + (q − 1)k−j � (a j 1 + · · · + a j m), for 0 ≤ j ≤ k − 1, is divisible by p s+b because p | n k−j , p s | a j 1 + · · · + a j m, and p b−1 | 1 k−j + · · · + (q − 1)k−j by Lemma 1. Also (q k − q)(a k 1 + · · · + a k m) is divisible by p s+b because p b | q − 1 | q k − q and p s | a k 1 + · · · + a k m. Thus p s+b divides our sum and our proof is complete. Remark. In fact, one can also show the converse statement: if {a1, a2, . . . , am} is as defined in the problem and a k 1 + a k 2 + · · · + a k m is divisible by m for every positive integer k, then every prime that divides m also divides n. The problems on this page are copyrighted by the Mathematical Association of America's American Mathematics Competitions. AMC logo.png

2018 USAMO (ProblemsResources)
Preceded by
Problem 2
Followed by
Problem 4
1 2 3 4 5 6
All USAMO Problems and Solutions